Tài liệu ôn thi học sinh giỏi Toán Lớp 11 - Bài tập dãy số giới hạn số 2 - Ngô Tùng Hiếu

docx 19 trang nhungbui22 11/08/2022 2120
Bạn đang xem tài liệu "Tài liệu ôn thi học sinh giỏi Toán Lớp 11 - Bài tập dãy số giới hạn số 2 - Ngô Tùng Hiếu", để tải tài liệu gốc về máy bạn click vào nút DOWNLOAD ở trên

Tài liệu đính kèm:

  • docxtai_lieu_on_thi_hoc_sinh_gioi_toan_lop_11_bai_tap_day_so_gio.docx

Nội dung text: Tài liệu ôn thi học sinh giỏi Toán Lớp 11 - Bài tập dãy số giới hạn số 2 - Ngô Tùng Hiếu

  1. 1. XÁC ĐỊNH SỐ HẠNG TỔNG QUÁT 1.1. DỰ ĐOÁN SỐ HẠNG TỔNG QUÁT VÀ CHỨNG MINH BẰNG QUY NẠP u1 11 Bài 1. Cho dãy số un xác định bởi : . Xác định số hạng tổng quát của un 1 10un 1 9n,n N dãy đã cho. Hướng dẫn giải Ta có: u1 11 10 1 u2 10.11 1 9 102 100 2 u3 10.102 1 9.2 1003 1000 3 n Dự đoán: un 10 n 1 Chứng minh theo quy nạp ta có 1 k u1 11 10 1, công thức 1 đúng với n 1. Giả sử công thức 1 đúng với n k ta có uk 10 k . k k 1 Ta có: uk 1 10 10 k 1 9k 10 k 1 Công thức 1 đúng với n k 1 n Vậyun 10 n , n N. u1 2 Bài 2. Cho dãy số (un ) biết . Xác định số hạng tổng quát của dãy. un 3un 1 1,n 2 Hướng dẫn giải 1 3 1 1 u 3u 1 u 3u u 3(u )(1) n n 1 n 2 n 1 2 n 2 n 1 2 1 1 5 Đặt v u v u n n 2 1 1 2 2 (1) vn 3vn 1,n 2 Dãy (vn ) là cấp số nhân với công bội là q 3. 5 Nên v v .qn 1 .3n 1 . n 1 2 1 5 1 Do đó u v 3n 1 ,n 1,2, n n 2 2 2 1.2. DÃY SỐ CHO BỞI CÔNG THỨC TRUY HỒI 1.3. PHƯƠNG TRÌNH ĐẶC TRƯNG 1.4. PHƯƠNG PHÁP DÃY SỐ PHỤ 1.5. DÃY SỐ SINH BỞI PHƯƠNG PHƯƠNG TRÌNH 1.6. SỬ DỤNG PHÉP THẾ LƯỢNG GIÁC 1.7. CÁC DẠNG KHÁC 2. MỘT SỐ DẠNG TOÁN LIÊN QUAN ĐẾN TÍNH CHẤT CỦA DÃY SỐ x1 1; x2 3 xn 6xn 1 xn 2 Bài 3. Cho dãy xn xác định bởi: n N 2 Chứng minh rằng ta có 2xn 2 là số chính phương. Hướng dẫn giải:
  2. Dễ thấy xn Z n N,n 1 Ta có xn = 6 xn 1 xn 2 xn 3xn 1 3xn 1 xn 2 x 2 6x .x 9x 2 9x 2 6x .x x 2 n n n 1 n 1 n 1 n 1 n 2 n 2 x 2 6x .x x 2 x 2 6x .x x 2 n n n 1 n 1 n 1 n 1 n 2 n 2 x 2 6 x .x x 2 2 2 từ đó ta có n n n 1 n 1 x2 6x2 .x1 x1 8 x2 6x .x x2 8 0 n 1 n n 1 n (1) ' Vì nên phương trình (1) phải có nghiệm nguyên . Do đó (1) có phải là số chính phương 2 2 2 2 ' 9x (x 8) 8(x 1) k Tức là tồn tại sao cho n n n (2) Từ (2) ta suy ra k phải là số chẵn k 2m ;m N 2 2 2 2 8(xn 1) 4m 2xn 2 m k N an n 0 2x 2 2 Vậy n là số chính phương. Bài 4. Cho dãy với được xác định bởi: n a) Chứng minh an chia hết cho n với mọi giá trị nguyên dương của . a b n n a1 1;a2 2;a3 6;a4 12 b) Đặt n . Chứng minh tồn tại vô số số nguyên dương an 4 2an 3 an 2 2an 1 an ,n 1 n để 2015 là một ước củabn . Hướng dẫn giải a) Ta có b1 1; b2 1; b3 2; b4 3 Dễ thấy bn Fn với n 1;2;3;4 . Bằng quy nạp ta chứng minh dãy bn trùng với dãy Fn Thật vậy: Mệnh đề đúng với n 1;2;3;4 Giả sử mệnh đề đúng đến n 3 . Khi đó ta có: (n 4)bn 4 2(n 3)Fn 3 (n 2)Fn 2 2(n 1)Fn 1 nFn Dùng công thức của dãy Fibonaci : Fm 2 Fm 1 Fm ta dễ dàng biến đổi vế phải thành n 4 Fn 4 suy ra bn 4 Fn 4. Vậy mệnh đề đúng với n 4 , do đó nó đúng với mọi n nguyên dương. Điều đó chứng tỏ an luôn chia hết cho n với mọi n nguyên dương. b) Gọi rn là số dư của bn cho 2015 với n 1;2;3;4 Trước tiên ta chứng minh rn là một dãy tuần hoàn. Thật vậy: Ta có : bn 2 b n 1 bn rn 2  rn 1 rn (mod 2015) Vì có vô hạn các cặp (r1;r2 ),(r2 ;r3 ), (rn ;rn 1) nhưng chỉ nhận hữu hạn giá trị khác nhau nên tồn tại ít nhất hai phần tử của dãy trùng nhau. Ta giả sử là (rm ;rm 1) (rm T ;rm T 1) (với T là một số nguyên dương). Ta chứng minh (rn) tuần hoàn với chu kỳ T .
  3. ) Ta có : rm 2  rm 1 rm (mod 2015); rm T 2  rm T 1 rm T (mod 2015) rm 2  rm T 2 (mod 2015) rm 2 rm T 2 Tiếp tục như vậy ta chứng minh được: rm k rm T k với mọi k 0 (1) ) Ta có : rm 1  rm 1 rm (mod 2015); rm T 1  rm T 1 rm T (mod 2015) rm 1  rm T 1(mod 2015) rm 1 rm T 1 Bằng quy nạp ta chứng minh được: rm k rm T k với k 1;2; ;m 1 (2) Từ (1) và (2) suy ra r là một dãy tuần hoàn. n n 0 Bổ sung vào dãy bn phần tử bo 0 thỏa mãn b0 b1 b2 suy ra r0 0 . Khi đó dãy rn là dãy tuần hoàn bắt đầu từ phần tử đầu tiên r0 = 0. Do đó tồn tại vô số phần tử trong dãy rn bằng 0 .Như vậy câu b được chứng minh xong. 3. GIỚI HẠN CỦA DÃY SỐ 3.1. TÍNH GIỚI HẠN BẰNG ĐỊNH NGHĨA 1 a1 a a Bài 5. Cho dãy số an xác định bởi : 3 2 2a 2a 2 n n an 1 2 3an 4an 1 Chứng minh rằng với mọi số thực a 0 thì dãy an hội tụ. Tùy theo a , hãy tìm giới hạn của dãy an . Hướng dẫn giải 1 Nếu a 0 thì a 2 (do bất đẳng thức AM-GM) a 1 1 Nếu a 0 thì a 2 (do bất đẳng thức AM-GM) nên a 2 a a * Nếu a 1 thì a1 2 . Ta chứng minh: an 2, n ¥ Hiển nhiên a1 2 . 2.23 2.22 2 Giả sử a 2 a 2 k k 1 3.22 4.2 1 Vậy lim an lim 2 2 . a 0 * . Nếu thì a1 2 . Ta chứng minh an 2 n ¥ a 1 Rõ ràng a1 2 . Giả sử ak 2 . Ta chứng minh ak 1 2 3 2 2ak 2ak 2 2 ak 1 2 2 2 2ak ak 2 0 ( đúng) 3ak 4ak 1 Ta chứng minh an là dãy giảm, thật vậy : 3 2 a 2 1 a 2 an 2an an 2 n n n, an 1 an 2 2 0 3an 4an 1 3an 4an 1 ( do tử âm, mẫu dương vì
  4. 2 7 an 2 3 3an 4an 1 0 2 7 a n 3 2 7 Mà a 2 3a 2 4a 1 0 ) n 3 n n an giảm và bị chặn dưới an có giới hạn là L . 3 2 3 2 2an 2an 2 2L 2L 2 lim an 1 lim 2 2 3an 4an 1 3L 4L 1 L 2 an 2 L 1 Vậy lim an 2 . . Nếu a 0 thì a1 2 . Tương tự, ta có: 3 2 a 2 1 a 2 an 2an an 2 n n n, an 1 an 2 2 0 3an 4an 1 3an 4an 1 nên an tăng. Hơn nữa an bị chặn trên bởi 1, thật vậy 3 2 2ak 2ak 2 2 ak 1 1 2 1 ak 1 (2a 3) 0 3ak 4ak 1 Vậy an tăng và bị chặn trên an có giới hạn là L . an 1,n , an 1 an 0,n 2L3 2L2 2 L L 1 a 1 L 2 3L2 4L 1 n Vậy lim an 1 Tóm lại: + Nếu a 1 thì lim an 2 a 0 + Nếu thì lim an 2 a 1 + Nếu a 0 thì lim an 1. Bài 6. Cho dãy số xn được xác định bởi x1 0 1 2 3 2015 * x x L n ¥ n 1 n 2 3 2015 xn xn xn xn Tìm giới hạn của dãy nxn khi n , với là số thực cho trước. Hướng dẫn giải Dễ dàng chứng minh được xn 0,n 1 bằng qui nạp Ta có 2 1 2 1 2 1 2 xn 1 xn , n 1 xn 1 xn xn 2 2 xn 2 ; n 1 xn xn xn 2 2 2 2 Bởi vậy n N, n 2 thì xn xn 1 2 xn 2 4  x1 2 n 1 xn 1, n 2 và lim xn n * 1 2 3 2015 Với n N , đặt xn 1 xn tn trong đó tn 2 3  2015 xn xn xn xn
  5. t xn 1; n 2 0 tn 2 , với t 2 3  2014 2015 (1), suy ra xn 2 2 2 1 2 1 2 2tn xn 1 xn xn tn xn 2 tn 2 2xntn 2 . khi n . xn xn xn 2 b1 x1 Áp dụng định lý trung bình Cesaro cho dãy bn với 2 2 bn xn xn 1, n 2. b1 b2  bn ta có lim bn 2 suy ra lim limbn 2. n n n n 2 2 2 2 2 2 2 2 x xn xn 1 xn 1 xn 2  x2 x1 x1 b b  b n 1 Mà n 1 2 n suy ra lim . n 2 n n n xn 2 n 1 Thật vậy ta có thể chứng minh trực tiếp lim như sau (chứng minh định lý trung bình Cesaro) n 2 xn 2 2 2 2 Xét dãy cn : c1 x1 2; cn xn xn 1 2 với n 2,3 *  lim cn 0 nên  0 tồn tại m N sao cho cn ,  n m. n 2 Gọi M max ci  với 1 i m 1. 2 m 1 M 2 m 1 M m 1 M  Với  ở trên tồn tại m 1 thì m' hay .   m 2 Xét n max m,m'. ta có  n n m 1 n m 1 | ci | ci | ci | m 1 M  m 1 M   i 1 i m i 1 2 . o đó theo định n n n n n 2 m 2 2 n | c | nghĩa lim i 1 i 0 n n 2 2 2 2 2 2 2 2 x xn xn 1 xn 1 xn 2  x2 x1 x1 c c  c n 1 n 1 2 n 2 . suy ra lim . n 2 n n n xn 2 1 Nếu 2 thì n.x n.x 2 khi n n n 2 2 2 Nếu 2 thì n.xn xn .n.xn khi n 2 2 Nếu 2 thì n.xn xn .n.xn 0 khi n . 3.2. TÍNH GIỚI HẠN BẰNG CÁC CÔNG THỨC CƠ BẢN Bài 7. Cho dãy số un được xác định như sau u1 1 u u u 2 u 4 u 6 16,n * n 1 n n n n ¥ n 1 Đặt vn  , hãy tính limvn . i 1 ui 5 Hướng dẫn giải: * Dễ thấy un 0,n ¥ . Theo bài ra ta có
  6. 2 2 2 2 2 un 1 un 6un un 6un 8 16 un 6un 4 un 6un 4 1 1 1 Suy ra un 1 1 un 1 un 5 un 1 1 un 1 un 5 n 1 n 1 1 1 1 1 1 Do đó vn   i 1 ui 5 i 1 ui 1 ui 1 1 u1 1 un 1 1 2 un 1 1 2 Mặt khác, từ un 1 un 6un 4 ta suy ra un 1 6un . Kết hợp với u1 1 ta có n 1 * 1 un 6 ,n ¥ limun lim 0 . un 1 1 1 1 1 Từ đó ta có limvn lim 2 un 1 1 2 * 2 * Bài 8. Cho dãy số thực un với n ¥ thỏa mãn ln 1 un nun 1,n ¥ . n 1 nu Tìm lim n . n un Hướng dẫn giải: * 2 Với mỗi n ¥ , đặt fn x ln 1 x nx 1, x ¡ 2 2x x 1 Ta có f ' x n n 1 0 n 1 x2 1 x2 ' x 1 fn x 0 n 1 Do đó fn x là hàm tăng thực sự trên ¡ . fn 0 1 0 Ta có 1 1 fn ln 1 2 0 n n 1 Do đó !u ¡ sao cho f u 0 và 0 u . n n n n n Ta thấy lim un 0 n 1 2 u2 lim ln 1 un n 1 Do đó: n 2 lim nun lim 1 ln 1 un 1 n n 2 1 n 1 nu nln 1 un n 2 u2 Vậy lim lim lim nun ln 1 un n 1. n n n un un
  7. 2n 3 Bài 9. Cho dãy số a ,n 1 thỏa mãn a 1,a a ,n 2 và dãy b ,n 1 thỏa mãn n 1 n 2n n 1 n n bn  ai ,n 1. Chứng minh dãy bn có giới hạn và tìm giới hạn đó. i 1 Hướng dẫn giải: Ta có 2nan 2n 3 an 1 an 1 2 n 1 an 1 nan ,n 1. n Do đó bn  2 iai i 1 ai 1 2 1 n 1 an 1 i 1 1 Ta chứng minh bằng quy nạp rằng na ,n 1. n n Thật vậy: - Với n = 1, ta có a1 1 nên khẳng định đúng. 2 n 1 3 2n 1 1 - Giả sử khẳng định đúng với n n 1 . Ta có an 1 an , ta cần chứng minh 2 n 1 2n 2 n n 2n 1 1 1 2n 1 n 1 2n n 2n 2 n n n 1 n 1 4n2 4n 1 n 1 4n3 1 3n Bất đẳng thức cuối đúng nên khẳng định trên đúng với n 1 . Theo nguyên lí qui nạp thì khẳng định được chứng minh 1 Ta có 2 1 2 1 n 1 an 1 bn 2 n 1 Theo nguyên lí kẹp thì dãy bn có giới hạn và limbn 2 . 4 a1 Bài 10. Cho dãy số an thỏa mãn: 3 n 1,n ¥ 2 2 n 2 an n an 1 n 1 anan 1 Tìm lim an . Hướng dẫn giải 2 2 * n 2 n Dễ thấy an 0,n ¥ . Từ giả thiết ta có n 1 an 1 an * 1 1 Với mỗi n ¥ , đặt yn ta có y1 1 và an 4 2 2 1 2 1 2 2 n n 2 yn 1 n yn n 1 n 2 yn 1 n yn yn 1 2 yn 4 4 n 2 2 2 2 2 n 1 n 2 1 4 4n2 n 1 Do đó yn y1 2 an 2 n 1 n 1 3 n 1 n2 16 n2 n 1 Vậy lim an 4 . 3.3. TÍNH GIỚI HẠN BẰNG ĐỊNH LÍ KẸP
  8. 1 Bài 11. Tìm lim n n n! Hướng dẫn giải n Trước hết ta chứng minh bất đẳng thức : n! > ( ) n (*) ( n N*). 3 1 Bằng phương pháp qui nạp. Thật vậy : với n =1 , ta có 1 > (đúng). 3 k Giả sử (*) đúng với n = k tức là : k! > ( )k . Ta đi chứng minh (*) đúng với 3 n = k+1. k k 1 3 k 1 Ta có (k+1)! = k!(k+1) >( ) k (k+1) = ( )k+1. > ( )k+1 1 3 3 (1 )k 3 k Bất đẳng thức cuối này đúng vì : 1 k k(k 1) 1 k(k 1)(k 2) (k k 1) 1 (1+ )k =1+ + . + + . = k k 2! k 2 k! k k 1 1 1 1 2 k 1 1 1 1 1 = 1+1+ (1 ) + + (1 )(1 ) (1 ) 0 < < . n n! n 3 Vì lim = 0 . n n 1 Do đó theo định lý về giới hạn kẹp giữa ta suy ra: lim = 0 . n n n! 1 Vậy lim(2014 ) =2014 n n! x1 1; x2 2 5 Cho dãy số xn thoả mãn 2 x x n 1 ;n * n 2 2 ¥ 4 xn Tính I lim xn Từ giả thiết suy ra mội số hạng của dãy đều dương y1 0; y2 1 Đặt yn log2 xn , ta có dãy * 2yn 2 5yn 1 2yn ;n ¥
  9. z1 2, z2 1 z 2; z 1 1 2 2zn 2 5zn 1 zn Lại đặt yn zn 2 , ta có dãy * 2zn 2 5zn 1 zn ;n ¥ 1 Tìm được số hạng tổng quát của dãy là z 4. . n 2n Từ đó ta có lim yn 2 lim xn 4 . 3.4. CÁC DẠNG KHÁC x 2016 1 Bài 12. Tìm các giá trị thực của tham số m để dãy số (xn): m * có giới hạn hữu hạn. xn+1 2 n N 1+xn Hướng dẫn giải: *) m > 0 0 xn m n 1 m 2mx Xét hàm số: f (x) ta có f '(x) f(x) nghịch biến trên 0;m x2 1 (x2 1)2 Suy ra (x2n ),(x2n 1) đơn điệu và bị chặn. 2017 x1 x3 x5 + 0 m x1 x2 , x3 2016 x2 x4 x6 4m m f ( f (1)) 1, x 1 x 1 n N * m2 4 2 2017 2n a(1 b2 ) m Giả sử lim x a,lim x b a 1, (I) 2n 2n 1 2 b(1 a ) m a b (II) 3 a a m 1 (I) b a (III) 1 a m a Khi o m 2 hệ (I) có nghiệm duy nhất (xn) có giới hạn hữu hạn. 2017 Khi 2 m hệ (II) có nghiệm duy nhất lớn hơn 1 và hệ (III) có nghiệm thỏa mãn a b . Do đó 2016 lim x2n lim x2n 1 (xn ) không có giới hạn. 2017 x1 x3 x5 m 2017 2016 x1 x2 , x1 x3 2016 x2 x4 x6 lim x2n lim x2n 1 (xn ) không có giới hạn. * + m 2017 2016 xn 2016 n N limxn 2016
  10. x1 x3 x5 + m 2017 2016 x1 x2 , x1 x3 x2 x4 x6 lim x2n lim x2n 1 (xn ) không có giới hạn. *) m 0 tượng tự ta có 0 m 2 và m 2017 2016 . Bài 13. Cho số thực a, xét dãy số x được xác định bởi n n 1 3 xn 6xn 6 x1 a, xn 1 2 ,n 1,2, 3xn 9xn 7 Tìm tất cả các giá trị của a để dãy số có giới hạn hữu hạn, tìm giới hạn đó? Hướng dẫn giải: Với a 1thì xn 1,n 1nên lim xn 1 n 3 3 xn 1 1 xn 1 2 Với a 1 thì xn 1 2 , xn 2 2 ,n 2 3xn 1 9xn 1 7 3xn 1 9xn 1 7 3 3n 1 xn 2 xn 1 2 a 2 Do đó ,n 1 xn 1 xn 1 1 a 1 n 1 n 1 2 a 1 3 a 2 3 Từ đó, tính được xn n 1 n 1 ,n 1, a 2 3 a 1 3 3 Kết luận + a a 1 a 2 lim xn 2 2 n 3 + a a 1 a 2 lim xn 1 2 n 3 3 3 + a xn ,n 1 lim xn . 2 2 n 2 Bài 14. Cho hai dãy số dương a , b xác định bởi: a 3,b 2 và n n 0 n n 0 0 0 1 an 1 an bn 1 an 1 2 2 an 1 bn Với mọi n 0,1,2, Chứng minh rằng hai dãy trên hội tụ và tìm giới hạn của chúng. Hướng dẫn giải: 1 Ta chứng minh bằng quy nạp a tan ,b ,n 0,1,2, (*) . Thật vậy n n n 3.2 cos 3.2n 1 Với n 0 , ta có a 3 tan tan ,b 2 , vậy * đúng 0 0 0 3 3.2 cos 3.20 1 2 1 Với n 1, ta có a tan tan ,b , vậy * đúng 1 1 1 3 6 3.2 3 cos 3.21
  11. 1 Giả sử khẳng định đúng đến n k,k 1, tức là a tan ,b n n n 3.2 cos 3.2n 1 Ta chứng minh a tan ,b . Thật vậy. Từ 1 ta có n 1 n 1 n 1 3.2 cos 3.2n 1 sin 1 2sin cos sin2 cos2 1 a n n 1 n 1 n 1 n 1 n 1 3.2 3.2 3.2 3.2 3.2 1 a 2 2 n 1 cos cos sin 3.2n 3.2n 1 3.2n 1 2 sin n 1 cos n 1 sin cos tan 1 3.2 3.2 n 1 n 1 n 1 3.2 3.2 3.2 a tan n 1 n 1 cos sin 1 tan 3.2 cos n 1 sin n 1 cos n 1 sin n 1 n 1 n 1 n 1 3.2 3.2 3.2 3.2 3.2 3.2 3.2 1 1 Khi đó từ 2 , suy ra b2 a2 1 tan2 1 b n 1 n 1 n 1 n 1 3.2 cos2 cos 3.2n 1 3.2n 1 1 Như vậy theo nguyên lý quy nạp thì a tan ,b ,n 0,1,2, n n n 3.2 cos 3.2n 1 1 Do đó lim an lim tan tan 0 0; lim bn lim 1 n n n n 3.2 n cos cos0 3.2n Kết luận: lim an 0; lim bn 1.■ n n Bài 15. Cho dãy số (un ) xác định như sau : u1 2014 2 2 un 1 un (1 2a)un a ;n 1,2, Tìm điều kiện của a để dãy số (un ) có giới hạn hữu hạn khi n và tính giới hạn đó. Hướng dẫn giải 2 Ta có: un 1 un (un a) 0 un 1 un ; n 1,2,3, * Suy ra dãy số (un ) tăng knn ; từ đó dãy số (un ) có giới hạn hữu hạn khi và chỉ khi dãy bị chặn trên. 2 2 Giả sử lim un L (L ¡ ) , thì chuyển qua giới hạn hệ thức un 1 un (1 2a)un a ta có: n L L2 (1 2a)L a2 L a * - Nếu có chỉ số k ¥ mà uk a thì un a; n k trái với kết quả lim un L a n 2 2 Do đó: uk a với mọi k 1,2, hay un (1 2a)un a a, n 1,2,3, a 1 u1 a a 1 2014 a * Đảo lại: Nếu a 1 2014 a a 1 u1 a 2 2 (u1 a 1)(u1 a) 0 u1 (1 2a)u1 a a 0 u2 a và u1 u2 a 1 u2 a Bằng quy nạp ta chứng minh được a 1 un a, n 1,2,3, (H/s trình bày ra) Như vậy dãy (un ) tăng knn, bị chặn trên bới a , do đó dãy số (un ) có giới hạn hữu hạn.
  12. Kết luận: Với điều kiện a 1 2014 a thì dãy số (un ) có giới hạn hữu hạn khi n và lim un a . n x1 a 3 Bài 16. Cho dãy số xn thỏa mãn 2x x n ,n 1,2,3, n 1 2 3xn 1 Tìm a sao cho dãy số xác định và có giới hạn hữu hạn. Hướng dẫn giải 2x3 3 Đặt f x , x . Ta có x a, x f x . Ta có 3x2 1 3 1 n 1 n 2 2 6x4 6x2 6x x 1 f ' x 2 2 . 3x2 1 3x2 1 Bảng biến thiên x 3 3 -1 0 1 3 3 f’(x) -1 0 f(x) 1 3 Ta xây dựng dãy số như sau a , a f a , a f a , a f a , . 0 3 0 1 1 2 2 3 Nhận thấy a1,a3 , ,a2k 1, 0; a0 ,a2 , ,a2k , 0 . 3 3 Dựa vào bảng biến thiên ta thấy a ;0 ,a f 1 a 0; 1 2 1 3 3 a2 a0 f a3 f a1 a3 a1 f a4 f a2 a4 a2 . 3 Bằng quy nạp ta chứng minh được dãy a đơn điệu giảm, bị chặn bởi 0 và , dãy a đơn điệu 2k 3 2k 1 3 tăng và bị chặn bởi và 0. Từ đó tồn tại lim a2k , lim a2k 1 . 3 k k Ta có an f an 1 f f an 2 lim an f f lim an 2 l f f l
  13. 3 2l3 2 2 3l 1 2 1 2 4 2 l 2 l l l 1 20l 15l 5 0 (*) 2l3 5 3 2 1 3l 1 2x3 3 3 3 (do f x , x liên tục trên ;0 , 0; và l lim an ). 2 n 3x 1 3 3 3 3 1 3 5 Xét 0 l . Ta có f f an an an 2 an 0 nên * an . Vậy l . 3 5 3 5 5 Tương tự ta chứng minh được dãy a đơn điệu tăng, hội tụ về . 2k 1 5 5 nÕu n ch½n 5 5 +) Nếu a thì x2 x1, x3 x2 nên ta có dãy xn 5 5 nÕu n lÎ 5 Dãy này không hội tụ. 5 nÕu n ch½n 5 5 +) Nếu a ta có dãy xn 5 5 nÕu n lÎ 5 Dãy này không hội tụ. +) Nếu tồn tại n sao cho a an thì ta có 3 x a f x f a x a f x f a x a , , x a 1 n 1 n 2 n 1 2 n 1 3 n 2 n 1 0 3 Khi đó không tồn tại xn 2 . Vậy nếu a an thì dãy không xác định. 5 +) Nếu 0 a thì hai dãy con x , x cùng hội tụ về 0 nên giới hạn của dãy là 0. 5 2k 2k 1 Nếu a 1 thì x2 f a a x1 và hàm số đồng biến nên dãy đơn điệu giảm, bị chặn dưới bởi 1. Khi đó dãy hội tụ về 1. 3 +) Nếu a 1 thì x f a 1. Khi đó ta có thể khảo sát dãy từ x . Trường hợp này dãy đơn điệu 3 2 2 giảm và bị chặn dưới bởi 1 nên hội tụ về1 . +) Nếu a = 1 thì xn 1 n nên dãy hội tụ về 1.
  14. 5 3 5 3 +) Nếu a ta có lim a2n và a0 nên tồn tại a2k ,a2k 2 sao cho a2k 2 a a2k (Thật 5 3 5 n 3 3 vậy, các số hạng của a không thể cùng nằm bên trái a do a , chúng cũng không thể cùng nằm 2k 0 3 3 5 bên phải a do nếu thế thì a a2n lim a2n ). 3 n 5 3 Vậy a a ;a x a ;a , , x a ;a , x a ; x . Khi đó ta lại có dãy 2k 2 2k 2 2k 2k 2 2k 2 0 2k 2 0 2k 2 3 đơn điệu giảm, bị chặn dưới bởi 1 nên hội tụ về 1. 5 3 Vì f(x) là hàm lẻ nên trường hợp a 0, 1 a , a 1, a 1 ta khảo sát tương tự. 5 3 Kết luận: Điều kiện để dãy xác định và có giới hạn hữu hạn là 3 5 a ; a ; a a ,n 1,2,3, 3 5 n n Bài 17. Cho dãy số an xác định bởi 0 a1 1 và an 1 an ,n 1. an Chứng minh rằng lim an n 0 . n Hướng dẫn giải 1 Áp dụng bất đẳng thức AM-GM ta có a2 a1 2 (do a1 1) a1 Nhận xét: an n,n 2 . Ta sẽ chứng minh nhận xét này bằng phương pháp quy nap. Thật vậy • Với n 2 ta có a2 2 (đúng) • Giả sử ak k k 2 • Ta có ak 1 ak k 1 ak k k 1 ak ak 2 ak k 1 ak k 0 ak 1 ak k 0 (đúng) Suy ra ak 1 k 1 Như vậy an n,n 2 (điều phải chứng minh). n n Mặt khác, an 1 n 1 an n 1 an n 1 an an a2 n 1 a n a n a 1 n n n n (1) an an
  15. Áp dụng (1) ta có a2 2 a2 1 a3 3 a 2 a3 3 a3 1 a4 4 a3 an n an 1 an 1 n 1 an a2 2 a2 1 a3 3 a3 1 an n an 1 Suy ra a3 3 a4 4 an 1 n 1 a2a3 an a2 2 a2 1 a3 1 an 1 an 1 n 1 . a2a3 an 1 1 1 an 1 n 1 a2 2 1 1 1 a2 a3 an n 1 an 1 n 1 a2 2  1 (2) i 2 ai n an 1 1 an 1 1 an an n Ta lại có 1 (do an n 1) an 1 an 1 an 1 an 1 an n 1 a a a a Suy ra  1 1 . 2 n 1 1 . i 2 ai a2 a3 an an a1 a1 Từ (2) an 1 n 1 a2 2 . a2 2 . (vì an n ) an n a 0 a n 1 a 2 . 1 . n 1 2 n a1 a1 Mà lim 0 lim a2 2 0 . n n n n Do đó lim an 1 n 1 0 hay lim an n 0 . n n * Bài 18. Cho p ¥ , a 0 và a1 0 . Xét dãy số (an ) được xác định bởi: 1 a an 1 ( p 1)an p 1 , với mọi n 1. p an Chứng minh dãy số (an ) có giới hạn hữu hạn khi n . Hãy tìm giới hạn đó. Hướng dẫn giải * Theo bất đẳng thức Côsi ta có: 1 a 1 p 1 a p a a a a p. p a . a , với  n 1. (1) n 1 p n nn a p 1 p n a p 1 p 1 n n
  16. 1 a an 1 an ( p 1)an p 1 an p a Do đó: n p an a an a p 1 p 1 0;  n 2 (2) p p.an p.an p Từ (1) và (2) ta có dãy số (an ) giảm và bị chặn dưới bởi a ; suy ra dãy số (an ) có giới hạn hữu hạn khi n . p Giả sử lim an L ; ( L a ). n 1 a Chuyển qua giới hạn hệ thức an 1 ( p 1)an p 1 p an 1 a ta có phương trình L ( p 1)L pLp ( p 1)Lp a p Lp 1 Lp a L p a (thỏa mãn điều kiện). p Vậy lim an a . n Bài 19. Cho trước số thực dương và xét dãy số dương xn thỏa mãn 1 1 * xn 1 1 với mọi n ¥ . xn Chứng minh rằng dãy xn hội tụ và tìm giới hạn của nó. Hướng dẫn giải 1 Xét hàm số f (x) x , x 0 . x 1 1 1 x 1 Ta có f '(x) x 1 ; f '(x) 0 x x 1 . x2 x2 0 Ta có bảng biến thiên của hàm f(x): x 0 x0 +∞ f'(x) 0 + +∞ +∞ f(x) f(x0) 1 1 1 1 Suy ra f (x) f x0 ( 1) 1 1 1 Do đó xn 1 1 xn 1 xn xn 1 Suy ra xn 1 xn hay xn là dãy giảm. Kết hợp với xn 0 với mọi n ta suy ra dãy xn hội tụ. 1 Đặt lim x  0 . Chuyển qua giới hạn ta được  ( 1) 1  x n  0 1 1 Vậy lim xn . Bài 20. Tìm tất cả các hằng số c 0 sao cho mọi dãy số dãy số (un ) thỏa mãn
  17. un (0;1) n 1 un 1(1 un ) c đều hội tụ. Với giá trị c tìm được hãy tính giới hạn của dãy (un ) . Hướng dẫn giải Ta xét các trường hợp sau 1 c cun + Nếu c , thì từ giả thiết, ta có un 1 4cun ; n 1 4 1 un un (1 un ) 1 Từ đây bằng quy nạp, ta suy ra u (4c)n 1u . Do 4c 1 nên u khi n . Do đó, c n 1 n 4 không thỏa mãn 1 1 1 4c 1 1 4c a(1 b) c + Nếu 0 c , thì tồn tại a,b ; , a b sao cho . Thật vây, lấy 4 2 2 b(1 a) c 1 1 4c 1 1 4c a ; , đặt b a x (x 0) , thì 2 2 a(1 a) c a(1 b) c a(1 a x) c x . a Chú ý là b(1 a) a(1 a) c. Do đó, ta chỉ cần chọn x 0 như trên và b a x, thì được 2 bất đẳng thức nêu trên. Xét dãy số (un ) xác định bởi a khi n 2m un b khi n 2m 1 1 thì dãy (u ) thỏa mãn giả thiết nhưng không hội tụ. Thành thử, 0 c cũng không thỏa mãn. n 4 1 1 un + Nếu c , thì un 1 un . Suy ra dãy (un ) tăng và bị chặn. Do đó, (un ) hội tụ. 4 4(1 un ) 4un (1 un ) 1 1 1 Đặt x limu , thì từ giả thiết ta có x(1 x) hay x . Vậy limu . n 4 2 n 2 2 * Bài 21. Cho dãy số un xác định như sau: u1 2 , un 1 un un 1, n ¥ . Tìm giới hạn của dãy sn 1 1 1 * với sn , n ¥ . u1 u2 un Hướng dẫn giải Bằng phép quy nạp đơn giản ta thấy rằng: un 2 . 2 Xét tính đơn điệu của dãy un . Từ hệ thức un 1 un un 1 ta suy ra được * 2 n ¥ ,un 1 un un 1 0 , vậy dãy số un tăng. Tính tổng: Từ hệ thức truy hồi (1) ta suy ra được un 1 1 un un 1 1 1 1 1 1 1 1 * với n ¥ * un 1 1 un un 1 un 1 un un un 1 un 1 1 Thay n bởi 1, 2, 3, , n vào (*) và cộng vế với vế các đẳng thức ta suy ra :
  18. 1 1 1 1 1 u1 u2 un un 1 1 Do dãy un là dãy tăng nên có hai khả năng sau xảy ra: 1) Dãy un bị chặn trên. Theo tiêu chuẩn Weierstrass, nên un tăng và bị chặn trên nên nó có giới hạn. Giả sử lim un a a 2 . Chuyển qua giới hạn hệ thức (1) khi n ta có: n a a2 a 1 a2 2a 1 0 a 1, vô lý. 2) Dãy không bị chặn trên, do un tăng và không bị chặn trên nên 1 lim un lim un 1 lim 0 n n n un 1 1 1 1 Vì thế từ (2) ta suy ra: lim lim 1 1 n n u1 u2 un un 1 Bài 22. Cho dãy số (un) thỏa mãn : u0 2016;un 1 un 2 . un u3 Tính lim n . n n Hướng dẫn giải 3 3 1 3 3 1 (un 1) un 2 un 3 3 6 un un un 3 3 3 1 3 Do un 0 n => (un 1) un 3 3 6 un 3 ,n un un 3 3 3 suy ra (un ) u0 3n 2016 3n, n ¥ (1) Lại có 3 3 1 3 3 1 (un 1) un 2 un 3 3 6 un un un 3 3 1 3 1 1 un 3 3 2 un 3 2 2016 3n 20163 3n n 3n 3 3 1 1 => (un 1) un 3 n ¥ n 3n 2 Suy ra n 1 1 n 1 1 n 1 n 1 (u )3 u3 3(n 1) u3 3n n 1   2 1   2 k 1 k k 1 9k k 1 k k 1 9k n 1 1 1 1 1 Do 1 2 2  2 k 1 k 1.2 2.3 (n 1)n n 2 n 1 n 1 và n 2n (Bất đẳng thức Bunhiacopxki)   2 k 1 k k 1 k 2 suy ra (u )3 u3 3n 2n (2) n 1 9 Từ (1) và (2) suy ra
  19. 2 20163 3n (u )3 u3 3n 2n , n ¥ n 1 9 20163 (u )3 u3 2 2 3 n 1 3 , n ¥ n n n 9n n u3 Do đó lim n 3 n n Bài 23. Cho số thực a, xét dãy số xn xác định bởi: x1 a, xn 1 ln 3 cos xn sin xn 2014, n 1,2 Chứng minh rằng dãy số trên có giới hạn hữu hạn khi n . Hướng dẫn giải Đặt f x ln 3 sin x cos x 2014,x ¡ cos x sin x f ' x 3 sin x cos x 3 f ' x 2 cos x 2 f ' x sin x 4 4 . 2 2 2 9 f ' x 2 2 f ' x f ' x q,x ¡ 7 Áp dụng định lí Lagrange cho hàm số f x liên tục và có đạo hàm trên ¡ , thì với mọi số thực x,y tồn tại z ¡ sao cho: f x f y f ' z x y q x y f x f y q x y ,x, y ¡ * m n 1 Với m n m,n ¥ , ta có: xm xn f xm 1 f xn 1 q xm 1 xn 1 q xm n 1 x1 * Mặt khác: 2014 xn 2014 ln 5,n ¥ xn bị chặn * m n 1 Do đó:  0,N ¥ : q xm n 1 x1 ,m n N. Vậy xn là dãy Cauchy, nên dãy số đã cho hội tụ. 4. GIỚI HẠN CỦA HÀM SỐ 4.1. TÍNH GIỚI HẠN BẰNG ĐỊNH NGHĨA 4.2. TÍNH GIỚI HẠN BẰNG CÁC CÔNG THỨC CƠ BẢN 4.3. TÍNH GIỚI HẠN BẰNG ĐỊNH LÍ KẸP 4.3. TÍNH GIỚI HẠN BẰNG ĐẠO HÀM 4.4. CÁC DẠNG KHÁC